[Date Prev][Date Next][Thread Prev][Thread Next][Date Index][Thread Index]

[obm-l] Re: [obm-l] Análise Real



Bernardo,

  Boa tarde,

  Só dois comentários:

  (1) Há algo "estranho" com o "corolário", ele é completamente trivial,
mas não sei como concluir do exercício original esse resultado. Veja o
seguinte, Q não pode ser a renuião enumerável de abertos, simplesmente
porque cada aberto não vazio de R contém um inervalo aberto (a,b) não
vazio. Logo se Q fosse uma reunião de abertos (enumerável ou não) Q
conteria (a,b). Iso é absurdo pois R-Q é denso em R. Talvez o corolário
seja "Q não é a intercecção enumerável de abertos". De fato isso segue-se
imediatamente do exercício proposto, por passagem ao complementar.

  (2) Não sei exatamente o contexto em que o exercício apareceu, às vezes
quando se fala em R, esconde-se quando se está usando Baire. Você precisa
saber alguma coisa, por exemplo que R não pode ser escrito como reunião
enumerável de fechados sem interior [pode chamar isso propriedade de Baire
da reta] ou algo equivalente para fazer o exercício (o que foi usado na
demonstração do outro email foi algo equivalente). Se você souber dessa
propriedade que enunciei acima, uma demonstração "alternativa" (que, no
fundo é exatamente igual) é a seguinte.

    Suponha, por absurdo, que existem subconjuntos fechados de R, F_1,
F_2,..., F_n,... tais que a reunião de todos os F_n seja R-Q.

    Como Q não tem interior (pois nenhum intervalo aberto da reta, não
vazio, está contido em Q) segue-se que cada F_n tem interior vazio.

   Qomo Q é enumerável tome {q_k, k em N} uma enumeração de Q e defina
T_j={q_j}, j=1,2,...

  Claro que cada T_j é fechado e de interior vazio.

  Então R = (R-Q) U Q seria a reunião dos F_n com os T_j. Então ter-se-ia
escrito R como uma reunião enumerável de fechados sem interior, o que
contraria a aupramencionada propriedade de Baire da reta.

Manuel Garcia


 On Tue, 15 Jul 2003 bmat@zipmail.com.br wrote:

> Manuel,
> Boa tarde.
>
> Muito boa a solução para este problema, mas eu não conheço o teorema de
> Baire, nem lembro muito bem o que era um espaço de Baire. Mas o pior é que
> este problema tinha um "corolário": conclua que Q não é a reunião enumerável
> de abertos... então eu suponho que deve haver outro meio para resolver este
> problema. Para ser mais completo, deixo agora a referência:
> Curso de Análise, vol 1 - Elon Lages Lima
> Capítulo 5 (Topologia da Reta) - exercício 55
>
> Muito obrigado pela atenção,
> Bernardo
>

=========================================================================
Instruções para entrar na lista, sair da lista e usar a lista em
http://www.mat.puc-rio.br/~nicolau/olimp/obm-l.html
=========================================================================